\classheader{2012-11-01} Today we'll go back over some more basic material that not everyone has seen yet. Recall the definition of $L^p$ space: \begin{definition} Given a measure space $(X,\mu)$, and a function $f$ on $X$ such that $\int |f|^p\,d\mu<\infty$, we say that $f\in L^p(\mu)$. When there exists a $K$ such that $|f(x)|\leq K$ almost everywhere, we say that $f\in L^\infty(\mu)$. \end{definition} \begin{definition} A normed space is a vector space $V$ with a function $\|\cdot\|:V\to\R$ such that \begin{itemize} \item $\|f\|\geq 0$ for all $f$, with equality if and only if $f=0$ \item $\|cf\|=|c|\cdot\|f\|$ \item $\|f_1+f_2\|\leq\|f_1\|+\|f_2\|$ \end{itemize} We say that $V$ is complete if $\rho(f,g)=\|f-g\|$ is a complete metric. A complete normed space is called a Banach space. \end{definition} As we will see later, the norms \[\|f\|_p=\left(\int|f|^p\,d\mu\right)^{1/p},\qquad \|f\|_\infty=\inf\{K:|f(x)|\leq K\text{ a.e.}\}\] make $L^p$ and $L^\infty$, respectively, into complete normed spaces, but only after we identify functions $f$ and $g$ if $f=g$ a.e. (otherwise we can have $\|f\|=0$ even when $f\neq 0$). \begin{theorem}[H\"older's Inequality] For any $f\in L^p$ and $g\in L^q$ where $\frac{1}{p}+\frac{1}{q}=1$, and either $1,name path global=linethree] (-0.2,0) -- (4.2,0) node[right] {$x$}; \draw[->] (0,-1.2) -- (0,1.8) node[above] {$y$}; \draw[name path global=lineone,color=blue,domain=0.3:4] plot function{log(x)} node[right] {$\log(x)$}; \begin{scope} \clip plot[domain=0.3:4] function{log(x)}; \clip (0,0) rectangle (3,2); \draw [name path global=linetwo,color=red,domain=0:4] plot function{0.5*x-0.4}; \end{scope} \begin{scope} \clip (0,0) rectangle (4,2); \draw[name path global=linefour,densely dotted,name intersections={of=lineone and linetwo}] (intersection-1) node [fill,circle,minimum size=3pt,inner sep=0pt,outer sep=0pt,label={[label distance=1pt,inner sep=0pt,outer sep=0pt]135:{\small $\log(a)$}}] (fa) {} -- ++(0,-2); \draw[name path global=linefive,densely dotted,name intersections={of=lineone and linetwo}] (intersection-2) node [fill,circle,minimum size=3pt,inner sep=0pt,outer sep=0pt,label={[label distance=3pt,inner sep=0pt,outer sep=0pt]92:{\small $\log(b)\;$}}] (fb) {} -- ++(0,-3); \end{scope}' \begin{scope}[name intersections={of=linethree and linefour}] \node [fill,circle,minimum size=3pt,inner sep=0pt,outer sep=0pt,label={[label distance=3pt,inner sep=0pt,outer sep=0pt]270:{\small $a$}}] at (intersection-1) {}; \end{scope} \begin{scope}[name intersections={of=linethree and linefive}] \node [fill,circle,minimum size=3pt,inner sep=0pt,outer sep=0pt,label={[label distance=3pt,inner sep=0pt,outer sep=0pt]270:{\small $b$}}] at (intersection-1) {}; \end{scope} %\draw[color=blue,domain=0.3:4] plot function{log(x)} node[right] {$\log$}; \end{tikzpicture} \end{center} so this is true. \end{proof} \begin{proof}[Proof of H\"older] Let's do the case when $\|f\|_p=1$ and $\|g\|_q=1$ first. Let $\lambda=\frac{1}{p}$ and $1-\lambda=\frac{1}{q}$, and let $a=|f(x)|^p$ and $b=|g(x)|^q$. The lemma implies that \[|f(x)|\cdot|g(x)|\leq\frac{1}{p}|f(x)|^p+\frac{1}{q}|g(x)|^q,\] and therefore \[\int|f(x)|\cdot|g(x)|\leq\frac{1}{p}\underbrace{\int|f(x)|^p}_{=\,1}+\frac{1}{q}\underbrace{\int|g(x)|^q}_{=\,1}=\frac{1}{p}+\frac{1}{q}=1.\] In the general case, we can just let $F=\frac{f}{\|f\|_p}$ and $G=\frac{g}{\|g\|_q}$, so that we can apply the special case to see that \[\int|FG|\leq 1\] and hence \[\int\frac{|fg|}{\|f\|_p\|g\|_q}\leq 1\implies \int|fg|\leq\|f\|_p\|g\|_q.\] Finally, if $p=1$ and $q=\infty$, we have that \[\int|fg|\leq\int|f|\cdot K=K\int|f|\] when $|g|\leq K$ almost everywhere. \end{proof} \begin{theorem}[Minkowski Inequality] For any $1\leq p\leq \infty$, we have \[\|f+g\|_p\leq\|f\|_p+\|g\|_p\] for any $f,g\in L^p$. \end{theorem} \begin{proof} If $p=1$ or $p=\infty$, this is trivial. Now suppose $11\}$, we have that \[\int\limits_X|f|^p=\underbrace{\int\limits_{X_1}|f|^p}_{\leq \mu(X)<\infty}+\int\limits_{X_2}|f|^p\geq \int\limits_{X_2}|f|^\nu\] because $|f|^p\geq |f|^\nu$ on $X_2$, so that \[\int\limits_X|f|^\nu=\underbrace{\int\limits_{X_1}|f|^\nu}_{\leq \mu(X)<\infty}+\int\limits_{X_2}|f|^\nu\] is finite, and therefore $f\in L^\nu$. \end{proof} \begin{proposition} If $f\in L^\infty$, then $f\in L^p$ for any $p$. Moreover, $\|f\|_p\to\|f\|_\infty$ as $p\to\infty$. \end{proposition} \begin{proof} For any $t<\|f\|_\infty$, we have that $\mu(A)>0$ where $A=\{x\mid f(x)>t\}$. Then \[\int\limits_X|f|^p\geq\int\limits_A|f|^p\geq\int\limits_A t^p=t^p\mu(A).\] Thus $\|f\|_p\geq t\mu(A)^{1/p}$. As $p\to\infty$, we have that \[\liminf_{p\to\infty}\|f\|_p\geq t\cdot 1,\] and therefore $\liminf\|f\|_p\geq\|f\|_\infty$. In the other direction, $|f|\leq\|f\|_\infty$ a.e., so that \[\int|f|^p\leq\int\|f\|_\infty^p=\|f\|_\infty^p\cdot\mu(X),\] and hence $\|f\|_p\leq\|f\|_\infty\cdot\mu(X)^{1/p}$. Therefore, $\limsup_{p\to\infty} \|f\|_p\leq\|f\|_\infty\cdot 1$. \end{proof} \begin{homework} If $\mu(X)<\infty$, is it true that if $f\in L^p$ for all $10$, and let $A=\{x\mid f(x)>\delta\}$ where $\delta$ is chosen such that \[\int_{X/A}|f|^p<\frac{\epsilon}{4}.\] Choose an $n$ such that $A_n=\{x:f(x)\leq n\}$ satisfies \[\int_{X\setminus A_n}f^p<\frac{\epsilon}{4}.\] Lastly, choose $\eta$ such that \[\frac{\epsilon}{4\mu(A_n)}=\eta^p.\] Then, again breaking up the range of $f$, we define the set \[M_\nu=\{x\in A_n\mid (\nu-1)\eta\leq f(x)\leq \nu\eta\},\] and now we can define the simple function \[g(x)=\begin{cases} 0 & \text{ if }x\notin A_n,\\ (\nu-1) & \text{ if }x\in M_\nu. \end{cases}\] This satisfies \[\int|f-g|^p=\int_{X\setminus A}|f|^p+\sum_{\nu}\int_{M_\nu}|f-g|^p+\int_{A/A_n}|f|^p<\left(\frac{\epsilon}{4}\right)+\underbrace{\eta^p\cdot\mu(A_n)}_{=\,\epsilon/4}+\left(\frac{\epsilon}{4}\right)<\epsilon.\qedhere\] \end{proof} Next time, we'll look more at the special properties of $\frac{1}{p}+\frac{1}{q}=1$. In particular, we'll prove that the dual of $L^p$ is $L^q$ and the dual of $L^1$ is $L^\infty$.